Diễn Đàn MathScopeDiễn Đàn MathScope
  Diễn Đàn MathScope
Ghi Danh Hỏi/Ðáp Community Lịch

Go Back   Diễn Đàn MathScope > Sơ Cấp > Tài Liệu > Nơi Yêu Cầu Tài Liệu

News & Announcements

Ngoài một số quy định đã được nêu trong phần Quy định của Ghi Danh , mọi người tranh thủ bỏ ra 5 phút để đọc thêm một số Quy định sau để khỏi bị treo nick ở MathScope nhé !

* Nội quy MathScope.Org

* Một số quy định chung !

* Quy định về việc viết bài trong diễn đàn MathScope

* Nếu bạn muốn gia nhập đội ngũ BQT thì vui lòng tham gia tại đây

* Những câu hỏi thường gặp

* Về việc viết bài trong Box Đại học và Sau đại học


Trả lời Gởi Ðề Tài Mới
 
Ðiều Chỉnh Xếp Bài
Old 24-02-2011, 05:52 PM   #1
thanh_tung
+Thành Viên+
 
Tham gia ngày: Aug 2010
Bài gởi: 7
Thanks: 6
Thanked 10 Times in 3 Posts
Đề thi chọn học sinh giỏi lớp 10 trung học phổ thông chuyên sư phạm.

Em không biết gõ latex nên gửi đề lên vậy. Híc
[RIGHT][I][B]Nguồn: MathScope.ORG[/B][/I][/RIGHT]
 
File Kèm Theo
Kiểu File : doc Đề thi chọn hsg chuyên SP.doc (55.5 KB, 168 lần tải)
thanh_tung is offline   Trả Lời Với Trích Dẫn
The Following 5 Users Say Thank You to thanh_tung For This Useful Post:
hoanghai_vovn (24-02-2011), Phongvan34 (25-02-2011), Qdragon (24-02-2011), ronaldo_789x (24-02-2011), stupidboy (25-02-2011)
Old 24-02-2011, 06:11 PM   #2
ThangToan
+Thành Viên+
 
Tham gia ngày: Nov 2010
Đến từ: THPT chuyên Vĩnh Phúc
Bài gởi: 570
Thanks: 24
Thanked 537 Times in 263 Posts
Trường Đại học Sư phạm Hà Nội
TRƯỜNG THPT chuyên


ĐỀ THI CHỌN HỌC SINH GIỎI LỚP 10

Năm học: 2010-2011

MÔN TOÁN, NGÀY 1, THỜI GIAN: 180 PHÚT


Câu 1: Giải hệ phương trình:
$\left\{ \begin{array}{l}
17({x^2} + {y^2}) + \frac{{12}}{{{{(x + y)}^2}}} + 14xy = 521\\
2x + \frac{1}{{x + y}} = 12
\end{array} \right. $




Câu 2: Cho các số dương x,y,z,t. Tìm giá trị nhỏ nhất của biểu thức:
$\[S = \sum\limits_{cyc}^{x,y,z,t} {\sqrt[3]{{{{(\frac{x}{{x + y}})}^7}}}} \] $

Câu 3: Chứng minh rằng không tồn tại đa thức P(x) và Q(x) sao cho

$\sqrt {{x^2} + 1} = \frac{{P(x)}}{{Q(x)}} $ với mọi $x\in \mathbb{R} $
Câu 4: Cho tam giác ABC vuông tại A, đường cao AH. I,J theo thứ tự là tâm đường tròn nội tiếp tam giác ABH và tam giác ACH. HI và HJ theo thứ tự cắt AB,AC tại $C_1, B_1 $ . $BB_1, CC_1 $ theo thứ tự cắt IJ tại $B_2, C_2 $ .
1) Chứng minh rằng AH, $BB_1, CC_1 $ đồng quy và IJ song song $B_1C_1 $
2) Tính góc $B_2HC_2 $



.................................................. ..............Hết............................... ...............................
------------------------------
Câu 2: Cho các số dương x,y,z,t. Tìm giá trị nhỏ nhất của biểu thức:
$\[S = \sum\limits_{cyc}^{x,y,z,t} {\sqrt[3]{{{{(\frac{x}{{x + y}})}^7}}}} \] $

Trước hết ta chứng minh bổ đề:
Cho $a, b, c, d $ là các số dương thỏa mãn $abcd=1 $. Khi đó $\frac{1}{(1+a)^2}+\frac{1}{(1+b)^2}+\frac{1}{(1+c) ^2}+\frac{1}{(1+d)^2}\ge 1 $.
sau đó đặt $a=\frac{y}{x}, b=\frac{z}{y}; b=\frac{t}{z}, b=\frac{x}{t} $ thì $abcd=1 $. Sau đó sử dụng bdt AM-GM cho 6 số $m^{\frac{7}{3}} $ và số ${(\frac{1}{2})}^{\frac{7}{3}} $ ta được:
$6m^{\frac{7}{3}}+{(\frac{1}{2})}^{\frac{7}{3}}\ge
7m^2.{(\frac{1}{2})}^{\frac{1}{3}}.\sqrt[7]{6} $. Khi đó áp dụng với $m=a, b, c, d $ và sử dụng bổ đề trên ta được dpcm
[RIGHT][I][B]Nguồn: MathScope.ORG[/B][/I][/RIGHT]
 

thay đổi nội dung bởi: ThangToan, 24-02-2011 lúc 06:30 PM Lý do: Tự động gộp bài
ThangToan is offline   Trả Lời Với Trích Dẫn
The Following 10 Users Say Thank You to ThangToan For This Useful Post:
discovermath (25-02-2011), ductho (24-02-2011), hoanghai_vovn (24-02-2011), hungtoandc3 (06-03-2011), khaitang1234 (25-02-2011), n.v.thanh (24-02-2011), Qdragon (24-02-2011), ronaldo_789x (05-03-2011), thanh_tung (24-02-2011), tuan119 (24-02-2011)
Old 24-02-2011, 08:13 PM   #3
daylight
+Thành Viên+
 
daylight's Avatar
 
Tham gia ngày: Dec 2009
Đến từ: Ha Noi
Bài gởi: 551
Thanks: 877
Thanked 325 Times in 188 Posts
Trích:
Nguyên văn bởi thangtoancvp View Post
[CENTER]Trường Đại học Sư phạm Hà Nội
TRƯỜNG THPT chuyên

Câu 3: Chứng minh rằng không tồn tại đa thức P(x) và Q(x) sao cho

$\sqrt {{x^2} + 1} = \frac{{P(x)}}{{Q(x)}} $ với mọi $x\in \mathbb{R} $
Giả sử tồn tại thỏa mãn $2 $ đa thức $P(x) $và
$Q(x) $ thỏa mãn đề bài thì ta có :

$[P(x)]^2=[Q(x)]^2(x^2+1) $

Điều này chứng tỏ $\deg P(x)=\deg Q(x)+1. $

Suy ra$ P(x)=Q(x)(ax+b)+R(x) ( \deg R(x) < \deg Q(x)) $
Từ giả thiết ta có :

$[Q(x)(ax+b)+R(x)]^2=[Q(x)]^2(x^2+1) $

$\Rightarrow [Q(x)]^2(ax+b)^2+2Q(x)R(x)(ax+b)+[R(x)]^2=[Q(x)]^2x^2+[Q(x)]^2 $

$\Rightarrow a^2Q[(x)]^2x^2+2ab[Q(x)]^2x+b^2[Q(x)]^2+2Q(x)R(x)(ax+b)+[R(x)]^2=[Q(x)]^2x^2+[Q(x)]^2 $


Như vậy hệ số của x có số mũ cao nhất ở 2 vế sẽ bằng nhau , chứng tỏ $a=1 $.

Suy ra

$2b[Q(x)]^2x+b^2[Q(x)]^2+2Q(x)R(x)(x+b)+[R(x)]^2=[Q(x)]^2 $

suy ra $b=0 $
Tức là $[Q(x)]^2=2Q(x)R(x)x+[R(x)]^2 $ (1)
Từ đó ta dễ thấy $Q(x)=R(x)(cx+d)+S(x) (\deg S(x)<\deg R(x)) $

$\Rightarrow 2(R(x)(cx+d)+S(x))R(x)x+[R(x)]^2=(R(x)(cx+d)+S(x))^2 $

Tương tư ta cũng có $c=2 $

Lúc này $2[R(x)(2x+d)+S(x)]R(x)x+[R(x)]^2=[R(x)(2x+d)+S(x)]^2 $

tức là : $[R(x)]^2=[R(x)]^2(2dx+d^2)+2R(x)S(x)(x+d)+[S(x)]^2 $

Đến đây ta phải có $d=0 $

Suy ra

$[R(x)]^2=2R(x)S(x)x+[S(x)]^2 $

Ta thấy nó giống (1) và như vậy nếu ta làm liên tục thì sẽ có:
$\deg R(x)>\deg R_0(x)>\deg R_1(x)>...>\deg R_n(x)=0 $ thì sẽ không tồn tại $S(x) $ ( Vô lí)
ta có điều phải chứng minh.
[RIGHT][I][B]Nguồn: MathScope.ORG[/B][/I][/RIGHT]
 
daylight is offline   Trả Lời Với Trích Dẫn
The Following 4 Users Say Thank You to daylight For This Useful Post:
hoanghai_vovn (24-02-2011), huynhcongbang (25-02-2011), plasa88 (25-02-2011), ronaldo_789x (05-03-2011)
Old 25-02-2011, 12:02 AM   #4
huynhcongbang
Administrator

 
huynhcongbang's Avatar
 
Tham gia ngày: Feb 2009
Đến từ: Ho Chi Minh City
Bài gởi: 2,413
Thanks: 2,165
Thanked 4,188 Times in 1,381 Posts
Gửi tin nhắn qua Yahoo chát tới huynhcongbang
Trích:
Nguyên văn bởi thangtoancvp View Post
Câu 1: Giải hệ phương trình:
$\left\{ \begin{array}{l}
17({x^2} + {y^2}) + \frac{{12}}{{{{(x + y)}^2}}} + 14xy = 521\\
2x + \frac{1}{{x + y}} = 12
\end{array} \right. $
Đặt $t = x+y \neq 0 $.
Hệ đã cho tương đương với:
$\left\{ \begin{array}{l}17(x+y)^2 + \dfrac{{12}}{{{{(x + y)}^2}}} -20xy = 521\\2x + \frac{1}{{x + y}} = 12 \end{array} \right. \Leftrightarrow \left\{ \begin{array}{l}17t^2 + \dfrac{{12}}{{{t^2}}} -20xy = 521\\2x + \dfrac{1}{t} = 12 \end{array} \right. $

Từ PT thứ hai, ta thấy rằng: $x = 6 - \frac{1}{2t} $ và $y = t - x = t - 6 +\frac{1}{2t} = \frac{2t^2-12t+1}{2t} $.
Thay vào PT ở trên, ta được:
$17t^2 + \dfrac{{12}}{{{t^2}}} -20(6-\frac{1}{2t})(\frac{2t^2-12t+1}{2t}) = 521 \Leftrightarrow 17t^2 + \dfrac{{12}}{{{t^2}}} -\frac{5(12t-1)(2t^2-12t+1)}{t^2} = 521 $.

Quy đồng và rút gọn, ta được PT sau:
$17t^4-120t^3+209t^2-120t+17=0 $.
Đây là phương trình đối xứng nên ta có thể giải tiếp theo công thức.
Chia hai vế cho $t^2 $ và đặt $u = t+\frac{1}{t} $.
$ 17(t^2+\frac{1}{t^2})-120(t+\frac{1}{t})+209=0 \Leftrightarrow 17(u^2-2)-120u+209=0 \Leftrightarrow u = 5 \vee u = \frac{35}{17} $.

-Với $u = 5 $, ta tính được $t =\frac{5 \pm \sqrt{21}}{2} $. Ta được hai nghiệm của hệ là:
$(\frac{19-\sqrt{21}}{4}, \frac{-9-\sqrt{21}}{4}),(\frac{19+\sqrt{21}}{4}, \frac{-9+\sqrt{21}}{4}), $

- Với $u = \frac{35}{17} $, ta tính được $t = \frac{35 \pm \sqrt{69}}{34} $. Ta được hai nghiệm của hệ là:
$(\frac{373+\sqrt{69}}{68}, \frac{-303+\sqrt{69}}{68}),(\frac{67-\sqrt{69}}{68}, \frac{-303-\sqrt{69}}{68}), $.

Vậy HPT đã cho có 4 nghiệm phân biệt như trên.
[RIGHT][I][B]Nguồn: MathScope.ORG[/B][/I][/RIGHT]
 

thay đổi nội dung bởi: huynhcongbang, 25-02-2011 lúc 12:21 AM
huynhcongbang is offline   Trả Lời Với Trích Dẫn
The Following 9 Users Say Thank You to huynhcongbang For This Useful Post:
bachzealot (28-02-2011), Conan Edogawa (27-02-2011), daylight (25-02-2011), discovermath (25-02-2011), hungtoandc3 (06-03-2011), long_chau2010 (27-02-2011), phantiendat_hv (25-02-2011), ronaldo_789x (05-03-2011), tuanphamme (27-02-2011)
Old 25-02-2011, 06:30 AM   #5
ThangToan
+Thành Viên+
 
Tham gia ngày: Nov 2010
Đến từ: THPT chuyên Vĩnh Phúc
Bài gởi: 570
Thanks: 24
Thanked 537 Times in 263 Posts
Câu 3: Chứng minh rằng không tồn tại đa thức P(x) và Q(x) sao cho

$\sqrt {{x^2} + 1} = \frac{{P(x)}}{{Q(x)}} $ với mọi $x\in \mathbb{R} $

Giả sử tồn tại hai đa thức $P(x), Q(x) $ thỏa mãn yêu cầu. Khi đó ta có: $P^2(x)=(x^2+1)Q^2(x), \forall x\in \mathbb{R} $ (1) và
$degP(x)=degQ(x)+1 $.
Từ đó dễ thấy: $P(i)=0; P(-i)=0 $ nên $P(x)=(x^2+1)P_1(x) $ thay vào (1) ta có:
$Q^2(x)=(x^2+1)P_1^2(x), \forall x\in \mathbb{R} $. Lí luận tương tự ta được tồn tại đa thức $Q_1(x) $ sao cho:
$Q(x)=(x^2+1)Q_1(x) $ thay trở lại đẳng thức trên ta được:
$P_1^2(x)=(x^2+1)Q_1^2(x), \forall x\in \mathbb{R} $.
Cứ tiếp tục lí luận như vậy và do bậc của $Q(x) $ là hữu hạn nên xảy ra hai khả năng:
TH1. tồn tại $Q_n(x) $ có $degQ_n(x)=0 $ thì $degP_n(x)=1 $ kiểm tra trực tiếp vào đẳng thức
$P_n^2(x)=(x^2+1)Q_n^2(x), \forall x\in \mathbb{R} $ không xảy ra.
TH2. tồn tại $Q_n(x) $ có $degQ_n(x)=1 $ thì $degP_n(x)=2 $ kiểm tra trực tiếp vào đẳng thức
$P_n^2(x)=(x^2+1)Q_n^2(x), \forall x\in \mathbb{R} $ không xảy ra.
[RIGHT][I][B]Nguồn: MathScope.ORG[/B][/I][/RIGHT]
 
ThangToan is offline   Trả Lời Với Trích Dẫn
The Following 2 Users Say Thank You to ThangToan For This Useful Post:
daylight (25-02-2011), ronaldo_789x (05-03-2011)
Old 25-02-2011, 08:02 AM   #6
kthptdc4
Banned
 
Tham gia ngày: Mar 2008
Bài gởi: 99
Thanks: 41
Thanked 71 Times in 27 Posts
Trích:
Nguyên văn bởi thangtoancvp View Post
[CENTER]

Câu 1: Giải hệ phương trình:
$\left\{ \begin{array}{l}
17({x^2} + {y^2}) + \frac{{12}}{{{{(x + y)}^2}}} + 14xy = 521\\
2x + \frac{1}{{x + y}} = 12
\end{array} \right. $
Viết hệ lại thành
$\left\{ \begin{matrix}
12\left[ {{\left( x+y \right)}^{2}}+\frac{1}{{{\left( x+y \right)}^{2}}} \right]+5{{\left( x-y \right)}^{2}}=521 \\
\left( x+y \right)+\frac{1}{x+y}+\left( x-y \right)=12 \\
\end{matrix} \right.
$

Đặt $x+y+\frac{1}{x+y}=a;\,\,x-y=b $
[RIGHT][I][B]Nguồn: MathScope.ORG[/B][/I][/RIGHT]
 
kthptdc4 is offline   Trả Lời Với Trích Dẫn
The Following 8 Users Say Thank You to kthptdc4 For This Useful Post:
daylight (25-02-2011), ductho (25-02-2011), F7T7 (02-03-2011), ha linh (05-03-2011), hungtoandc3 (06-03-2011), long_chau2010 (27-02-2011), ronaldo_789x (05-03-2011), Unknowing (25-02-2011)
Old 25-02-2011, 10:44 AM   #7
ThangToan
+Thành Viên+
 
Tham gia ngày: Nov 2010
Đến từ: THPT chuyên Vĩnh Phúc
Bài gởi: 570
Thanks: 24
Thanked 537 Times in 263 Posts
Trích:
Nguyên văn bởi kthptdc4 View Post
Viết hệ lại thành
$\left\{ \begin{matrix}
12\left[ {{\left( x+y \right)}^{2}}+\frac{1}{{{\left( x+y \right)}^{2}}} \right]+5{{\left( x-y \right)}^{2}}=521 \\
\left( x+y \right)+\frac{1}{x+y}+\left( x-y \right)=12 \\
\end{matrix} \right.
$

Đặt $x+y+\frac{1}{x+y}=a;\,\,x-y=b $
Bài toán này đã có trong đề thi học sinh giỏi lớp 9 của tỉnh Thái Bình năm 2009 hay 2010 (mình không nhớ rõ)
[RIGHT][I][B]Nguồn: MathScope.ORG[/B][/I][/RIGHT]
 
ThangToan is offline   Trả Lời Với Trích Dẫn
Old 25-02-2011, 01:09 PM   #8
thanh_tung
+Thành Viên+
 
Tham gia ngày: Aug 2010
Bài gởi: 7
Thanks: 6
Thanked 10 Times in 3 Posts
Câu 3: Mình làm thế này chẳng biết có đúng không:
Điều kiện Q(x) khác 0 với mọi x hay Q(x) vô nghiệm
Suy ra degQ(x)=2t. Ta có:
P(x)=Q(x)$\sqrt{x^{2}+1} $;
Q(x) vô nghiệm, $\sqrt{x^{2}+1} $ vô nghiệm nên P(x) vô nghiệm;
Suy ra degP(x)=2h. Ta có:
($P(x))^{2} $=($Q(x))^{2} $.($x^{2} $+1)
Suy ra deg( $\sqrt{x^{2}+1} $)=2(h-t).
Vậy $\sqrt{x^{2}+1} $=$a^{2n}_{2n} $+...+$a_{0} $.( $a_{2n} $$\neq $0)
Hay $x^{2} $+1=$(a^{2n}_{2n} $+...+$a_{0})^{2} $
degvt=2; degvp chia hết cho 4, mâu thuẫn
Vậy không tồn tại P, Q thỏa mãn đề bài.
Mãi mới gõ được một bài ra hồn!!!
[RIGHT][I][B]Nguồn: MathScope.ORG[/B][/I][/RIGHT]
 

thay đổi nội dung bởi: thanh_tung, 25-02-2011 lúc 01:19 PM Lý do: Tự động gộp bài
thanh_tung is offline   Trả Lời Với Trích Dẫn
Old 25-02-2011, 05:45 PM   #9
stupidboy
+Thành Viên+
 
Tham gia ngày: Sep 2010
Đến từ: ngoài Trái Đất
Bài gởi: 35
Thanks: 34
Thanked 25 Times in 10 Posts
Đề thi ngày 2. Thời gian: 180 phút
__________________________________________________

Câu I: Cho dãy số $F(n) $ xác định bởi công thức: $F_1 = F_2 = 1 $ và $F_{n+2}=F_{n+1}+F_{n} $
Chứng minh rằng $F_n + 1 $ không là số nguyên tố với mọi $n \ge 4 $.

Câu II: Tìm các số nguyên dương $x,y $ $(x > 1) $ và các số nguyên tố $p,q $ thỏa mãn $p^x -q^x = 2^y $.

Câu III: Với mỗi số nguyên $x $, kí hiệu $r(x) $ là số dư khi chia $x $ cho $10 $. Xét $A={1;2;...;9} $. Hỏi có hay không tồn tại tập con $B $ của $A $ sao cho $A = \{ r(x+y)| x,y \in B, x \neq y \} $
[RIGHT][I][B]Nguồn: MathScope.ORG[/B][/I][/RIGHT]
 
__________________
STUIPDBOY

thay đổi nội dung bởi: stupidboy, 25-02-2011 lúc 08:43 PM
stupidboy is offline   Trả Lời Với Trích Dẫn
The Following 3 Users Say Thank You to stupidboy For This Useful Post:
daylight (25-02-2011), ductho (25-02-2011), thanh_tung (25-02-2011)
Old 25-02-2011, 07:08 PM   #10
huynhcongbang
Administrator

 
huynhcongbang's Avatar
 
Tham gia ngày: Feb 2009
Đến từ: Ho Chi Minh City
Bài gởi: 2,413
Thanks: 2,165
Thanked 4,188 Times in 1,381 Posts
Gửi tin nhắn qua Yahoo chát tới huynhcongbang
Trích:
Nguyên văn bởi stupidboy View Post
Câu I: Cho dãy số $F(n) $ xác định bởi công thức: $F_1 = F_2 = 1 $ và $F_{n+2}=F_{n+1}+F_{n} $
Chứng minh rằng $F_n + 1 $ không là số nguyên tố với mọi $n \ge 4 $.
Ở câu này, ta cần chứng minh hai điều sau:

- Với mọi $n \equiv 1, 2 \pmod{3} $ thì $F_n $ là số lẻ.
- Với mọi $n \equiv 0 \pmod{3} $ thì $F_n $ chia 3 dư 2.

Cả hai điều này đều được chứng minh bằng quy nạp với những số hạng đầu là $F_1 = F_2 = 1 $.

Dễ thấy rằng khi đó $F_n + 1 $ hoặc chia hết cho 2 hoặc chia hết cho 3 nên không là số nguyên tố.
[RIGHT][I][B]Nguồn: MathScope.ORG[/B][/I][/RIGHT]
 
huynhcongbang is offline   Trả Lời Với Trích Dẫn
The Following User Says Thank You to huynhcongbang For This Useful Post:
Phongvan34 (25-02-2011)
Old 25-02-2011, 07:34 PM   #11
Phongvan34
+Thành Viên+
 
Phongvan34's Avatar
 
Tham gia ngày: Oct 2010
Bài gởi: 15
Thanks: 16
Thanked 7 Times in 4 Posts
Tiền bối nào giải giúp vãn bối bài 3 ngày 2 với !
Đề ra chẳng rõ chút nào.
[RIGHT][I][B]Nguồn: MathScope.ORG[/B][/I][/RIGHT]
 
Phongvan34 is offline   Trả Lời Với Trích Dẫn
Old 25-02-2011, 07:51 PM   #12
thanh_tung
+Thành Viên+
 
Tham gia ngày: Aug 2010
Bài gởi: 7
Thanks: 6
Thanked 10 Times in 3 Posts
Trích:
Nguyên văn bởi huynhcongbang View Post
Ở câu này, ta cần chứng minh hai điều sau:

- Với mọi $n \equiv 1, 2 \pmod{3} $ thì $F_n $ là số lẻ.
- Với mọi $n \equiv 0 \pmod{3} $ thì $F_n $ chia 3 dư 2.

Cả hai điều này đều được chứng minh bằng quy nạp với những số hạng đầu là $F_1 = F_2 = 1 $.

Dễ thấy rằng khi đó $F_n + 1 $ hoặc chia hết cho 2 hoặc chia hết cho 3 nên không là số nguyên tố.
$F_9 = 34 $ chia 3 dư 1 mà!!!
[RIGHT][I][B]Nguồn: MathScope.ORG[/B][/I][/RIGHT]
 
thanh_tung is offline   Trả Lời Với Trích Dẫn
Old 25-02-2011, 07:56 PM   #13
daylight
+Thành Viên+
 
daylight's Avatar
 
Tham gia ngày: Dec 2009
Đến từ: Ha Noi
Bài gởi: 551
Thanks: 877
Thanked 325 Times in 188 Posts
Trích:
Nguyên văn bởi stupidboy View Post
Câu II: Tìm các số nguyên dương $x,y $ $(x > 1) $ và các số nguyên tố $p,q $ thỏa mãn $p^x +q^x = 2^y $
nếu $p=q=2 $ thì $y=x+1 $. do đó ta xét trường hợp có ít nhất một số lẻ nhưng tổng lũy thừa $x $ của chúng là một số chẵn nên ta chỉ cần xét trường hợp $p $ và $q $ đều lẻ.
Nếu $x $ là số chẵn thì vế trái sẽ chia cho 4 dư 2 còn vế phải chia hết cho 4 vì $y >x+1>2 $
Nếu $x $ là số lẻ thì $VT=(p+q)(p^{x-1}-...+q^{x-1})=2^y $
trường hợp này ta sẽ loại luôn vì theo đề bài không xét nghiệm của phương trình $p+q=2^y $.
Vậy ta có nghiệm duy nhất $(p,q)=(2,2) $ và $y=x+1 ( x \in \mathbb{N^+} ) $
Trích:
Nguyên văn bởi stupidboy View Post
Câu III: Với mỗi số nguyên $x $, kí hiệu $r(x) $ là số dư khi chia $x $ cho $10 $. Xét $A={1;2;...;9} $. Hỏi có hay không tồn tại tập con $B $ của $A $ sao cho $A = \{ r(x+y)| x,y \in B, x \neq y \} $
Tồn tại : Ví dụ $x=5,y=6 $ thì $r(5+6)=1 $
[RIGHT][I][B]Nguồn: MathScope.ORG[/B][/I][/RIGHT]
 
daylight is offline   Trả Lời Với Trích Dẫn
The Following User Says Thank You to daylight For This Useful Post:
ductho (25-02-2011)
Old 25-02-2011, 07:56 PM   #14
thanh_tung
+Thành Viên+
 
Tham gia ngày: Aug 2010
Bài gởi: 7
Thanks: 6
Thanked 10 Times in 3 Posts
Trích:
Nguyên văn bởi stupidboy View Post
Đề thi ngày 2. Thời gian: 180 phút
__________________________________________________



Câu II: Tìm các số nguyên dương $x,y $ $(x > 1) $ và các số nguyên tố $p,q $ thỏa mãn $p^x +q^x = 2^y $.
Chán quá! Câu này tôi chép sai đề thành : Tìm các số nguyên dương $x,y $ $(x > 1) $ và các số nguyên tố $p,q $ thỏa mãn $p^x-q^y=2y $.
Đời đau rồi!!! Ngồi làm cả giờ không ra!!!
[RIGHT][I][B]Nguồn: MathScope.ORG[/B][/I][/RIGHT]
 

thay đổi nội dung bởi: thanh_tung, 25-02-2011 lúc 08:16 PM
thanh_tung is offline   Trả Lời Với Trích Dẫn
Old 25-02-2011, 08:07 PM   #15
daylight
+Thành Viên+
 
daylight's Avatar
 
Tham gia ngày: Dec 2009
Đến từ: Ha Noi
Bài gởi: 551
Thanks: 877
Thanked 325 Times in 188 Posts
Trích:
Nguyên văn bởi thanh_tung View Post
Chán quá! Câu này tôi chép sai đề thành : Tìm các số nguyên dương $x,y $ $(x > 1) $ và các số nguyên tố $p,q $ thỏa mãn $p^x+q^y=2y $.
Đời đau rồi!!! Ngồi làm cả giờ không ra!!!
Đề thế này thì càng dễ mà bạn, theo Bernoulli:
$q^y=[(q-1)+1]^y > (q-1)y+1 $

vậy $2|p $ suy ra $p=2 $ vậy $2^x+2^y=2y $ , theo quy nạp thì $2^y \ge 2y $

Vậy PT vô nghiệm
[RIGHT][I][B]Nguồn: MathScope.ORG[/B][/I][/RIGHT]
 
daylight is offline   Trả Lời Với Trích Dẫn
Trả lời Gởi Ðề Tài Mới

Bookmarks


Quuyền Hạn Của Bạn
You may not post new threads
You may not post replies
You may not post attachments
You may not edit your posts

BB code is Mở
Smilies đang Mở
[IMG] đang Mở
HTML đang Tắt

Chuyển đến


Múi giờ GMT. Hiện tại là 10:02 AM.


Powered by: vBulletin Copyright ©2000-2024, Jelsoft Enterprises Ltd.
Inactive Reminders By mathscope.org
[page compression: 113.98 k/130.86 k (12.89%)]